Download as pdf
Download as pdf
You are on page 1of 32
Chapter 7 System of Particles and Rotational Motion Solutions (Set-1) SECTION -A Answer (3) — 360%28 195 radis 60 = 2 (ay +0)t =2x12nx6 = 36m rad 2% 2 8 So, No of revolutions N= 2 = . Nap 18 Answer (2) 4 -( len) Kyotai = 2mv2| 1+ 2 |= 20x2 = 40 J toil = 3" | t+ P| Answer (3) __gsino = 2x08 =25 me? 7 14m, MR? Answer (1) 4 ype 4 yp? — 0 2 fy = 2MR? and Ip = AMR? = 10. = 12.5 kg mv oo 04 2 9 Answer (1) 2 up? f =2 ur’ Iotow sphere = 3 Answer (4) Ha 1512 + 128+9 at So, %]4=36Nm Answer (1) Perpendicular distance from origin is zero. Answer (1) Corporate Office : Aakash Tower, 8, Pusa Road, New Delhi-110005. Phone : 011-47623456 58 System of Particles and Rotational Motion Solutions of Assignment (Set-1) (Level) 9. Answer (1) ‘hoy 1 MR, = (MRP + AMR?) My M am Jo 10. Answer (3) ( =| =o =rxF 1. PxB 1 a 2 13, Me 44. Angular momentum 15. [mir] 46. Zero. Vectors are parallel to each other |AxB|=ABsing=0 = sind =0 o 17. Angular momentum (L) 1 dur® 1B. IFS 19. The moment of inertia of a rigid body about a given axis of rotation is the sum of the product of the masses of the various particles and square of their perpendicular distance from the axis of rotation, Sr 20. The centre of mass of the system lie at the mid-point of the line forming the particles. div? a die? +KE,, = dm? +t w+ 21. KE=KE, a 2 Lele 23, At the centroid ofthe triangle. 24, Right hand scrow nul 25, Angular acceleration 26. When the ice on the polar caps melts the moment of inertia / increases because the mass concentrated near the axis of rotation spreads out. As no external torque is acting on the system, (2n\ == (% L=tw= IF] = constant to decreases, hence T increases, so the duration ofthe day will increase Corporate Office : Aakash Tower, 8, Pusa Road, New Delhi-110005. Phone : 011-47623456 Solutions of Assignment (Set-1) (Level!) System of Particles and Rotational Motion MR* 27. Moment of inertia of dise |= Let the thickness be x and radius be r, and r, as the masses are equal Rr? d, = mr d, Age de ho tgs 4, bla 2 ie my. my, 28. Vem m,+™m, as they are moving in opposite directions therefore, ve sign shows that the velocity of the C.M Is in the direction of the velocity of 6 kg mass. Tig? die’ 28. KE waton = 3 ve 1) at KE, = Fhe, KE = Flos G = 1.25 kg m? Corporate Office : Aakash Tower, 8, Pusa Road, New Delhi-110005. Phone : 011-47623456 60 System of Particles and Rotational Motion Solutions of Assignment (Set-1) (Level) 31 32 ‘Angular acceleration a = 2— 2 Torque (5) = la = 1.25 « 75 327 Nm Moment of inertia of a rod about an axis passing through its centre of mass is given by MP hae 2 according to parallel axis theorem A Cc 1 : : eo = Ine +m(5} : : My — 7 P P CM = ot ml ———— sR aaa B D 3ml? _ Ami? 12 12 ley * mP 3 = $me Corporate Office : Aakash Tower, 8, Pusa Road, New Delhi-110005. Phone : 011-47623456 Solutions of Assignment (Set-1) (Level!) System of Particles and Rotational Motion 61 ‘There are 3 rings so the total moment of inertia of the system about yy’ is 3 1a |= 5 mex 2+ Sor _ Ime 2 2 33, Moment of inertia of soli sphere is given by /= Zr Using, @ - of = 200 2m x 2m = ane 18492 4Anctoe 5xae <1 a5% 1? = 8.88 x 10-4 Nm 34. L= lo, w= 10 rad s,1 L= 3% 10=30kg mst 3 second 0 x 3 = 60 kg mes 35. Moment of inertial of the ring about its diameter J, i8 given by MR* Ning = 100 MR = 200 Moment of inertia of the disc of the same mass M and radius R about an axis passing through its centre perpendicular to its plane is given by MR? _ 200 ° lage = p= AZ 100 g om Corporate Office : Aakash Tower, 8, Pusa Road, New Delhi-110005. Phone : 011-47623456 62 System of Particles and Rotational Motion Solutions of Assignment (Set-1) (Level) 36. Let the mass and radius of the solid sphere be m and r respectively Keo tannin = Env! 0 where vis the linear velocity of the solid sphere with which it is moving K.E. of rotatior Vig? ze (ii) 2 nr? Moment of inertia of solid sphere = = mr and w= ~ r Substituting these values in (i), we get Vint ye = Ag? of rotation = Lx 2mr? x = Lime’ i KE. of rotation = 5x Emr x = = (iy Dividing () by (li) we get KE ofttranslation 2) KE ofrotation ~ Smny? 5 37. The centre of mass (CM) of a body is a point where the entire mass of the body is supposed to be concentrated whereas the centre of gravity (CG) of a body is a point where the whole weight of the body is supposed to be concentrated. Centre of mass does not have to do anything with the gravity. It depends only on the distribution of mass of the body, 38. ()) The total forces, /e. the vector sum of the forces acting on the rigid body is zero, If the total force on the body is zero, then the total linear momentum of the body does not change with time. So the above equation gives the condition for the translational equilibrium of the body. (il) If the total torque acting on the rigid body vanishes i.e. the vector sum of the torques on the rigid body is zero, ‘The total angular momentum of the body does not change with time. So the above equation gives the Condition for the rotational equilibrium of the body. Corporate Office : Aakash Tower, 8, Pusa Road, New Delhi-110005. Phone : 011-47623456 Solutions of Assignment (Set-1) (Level-) ‘System of Particles and Rotational Motion 63 39, S.No. [Linear Motion Rotational motion about a fixed axis 1. | Displacement x Angular displacement @ 2. a de Velocty v= % Angular velocity w= 22 yve & a yore 3. ov Acceleration a = © Angular acceleration a ® 4. | Mass Moment of inertia | 5. | Force F= Ma Torque + = ke 6. | Work aw’ Fads 7. Jo? Kinetic energy kinetic energy k= “2 2 8, | Power P= Fv Power P= wo 9, | Linear momentum ‘Angular momentum L = fo 10, | Equations of transitory mation | Equations of rotational motion usat a= ap rat tay mints tol? ate fae ots Lat vou =208 ot f= 208 where the symbols have their | where the symbols have thelr usual usual meaning, meaning 11 | Linear momentum is conserved it | Angular momentum is conserved if no no external force acts on the | extemal torque acis onthe system. system do 40. Power isthe rate of work done P= ‘Small work done by the torque + acting on the particle in producing angular displacement d® is given by dW = 1d8 se at do “@__' the rate of change of angular displacement, which is the angular velocity hence P=] 1 L dest= = a Ee a 0 E, = Sou = 80 &=35 Corporate Office : Aakash Tower, 8, Pusa Road, New Delhi-110005. Phone : 011-47623456 64 System of Particles and Rotational Motion Solutions of Assignment (Set-1) (Level) 2 Apa sou= 2 1 angular momentum becomes > 42. On folding arms, r decrease hence as Re therefore, moment of inertia decreases on folding the arms. = Joris constant as no external torque is acting 1, = ho, as | decreases, « (increases) so as to maintain the same angular momentum. Hence the dancer spins faster on folding her arms. 43, Based on the conservation of linear momentum, 44. © is the vector which is given by (Ax). & lies in the plane perpendicular to both A and & (2-1) + j(-4-1) + (-1-2) -7-2j-3k oO 3k 2 unit vector fi in the direction of G is given by . i-2j-3k 4 Varo “Tal 2] - 3k) 45, Que 2 2a = 2x10x1 = 4 kg m? 3 3 akg lo * 200 = 800 kg m? s 46. () The total forces, ie. the vector sum of the forces acting on the rigid body is zero, Fis Frtuth =D If the total force on the body is zero, then the total linear momentum of the body does not change with time. So the above equation gives the condition for the translational equilibrium of the body. Corporate Office : Aakash Tower, 8, Pusa Road, New Delhi-110005. Phone : 011-47623456 Solutions of Assignment (Set-1) (Level!) System of Particles and Rotational Motion 65) (il) If the total torque acting on the rigid body vanishes i.e. the vector sum of the torques on the rigid body is zero, +h tity ‘The total angular momentum of the body does not change with time. So the above equation gives the condition for the rotational equilibrium of the body. 47. Angular momentum is given by L=FxB ‘We can oblain the three rectangular components of angular moment ie. L,, L, and L, as we did in the case of Torque. L bit hi+bk pe xisyjitzk pisnierk Using determinant method, we get + ; o\ ik _ . (Li+i+hk) =|x yz] = (yPa2e)P+(2P, -xP)]+(xR, ~yP,)2 PRP, Comparing the components of both sides, we get EWR) 0 Ib =(2P,- xP) (i) [kz =(xP, - yP.} Ai) ‘And from the above three equations we get if the particle is moving in y ~ z plane it has the angular momentum. Component only in x-direction je. L,#0,l,4L,=0 Similarly if the particle is moving in x-z plane L, #0, L, only L, #0, L,=L,=0 and if the particle is moving in x-y plane 48. In one dimension, the centre of mass is given by mx, + Myx, m,+m, In two dimensions Ky + Maks y _ MY + MeV In three dimensions mx POH 2M = ys a ae 49. The force applied on a rigid body may rotate it in three dimensions. In that case we shall have three components of torque, which can be obtained by using rxF Corporate Office : Aakash Tower, 8, Pusa Road, New Delhi-110005. Phone : 011-47623456 66 System of Particles and Rotational Motion Solutions of Assignment (Set) (Level!) fetiegienk axis yjrzk FaRi+A J+A e Using Determinant method i (ui+s,i+n) = af, )F + (2F, — xF,)) +(xF, yh me F Equating the three rectangular components on two sides, we get YE, 2F, ol =2F,-xF, Lil) ee) From the above three equations we can see that if the object is moving in the y ~ z plane t, # 0 and 52,20. Similarly if it is moving in x - z plane t#.0 and +, = t, = 0 and ifit is moving in x - y plane #0 and 1, =1,=0. 50, Consider an incined plane of height h, as shown in the figure, A Ost Let a body roll down this inclined plane. From A to B at A, potential energy of the body is mgh as it rolls down (according to the law of conservation of energy). Its potential energy changes into kinetic energy of rotation. So applying here the law of conservation of energy, we get KE. = RE 1 fe) SBM (Ee ‘The above expression gives the maximum velocity with which the centre of mass of the body reaches the bottom of the inclined plane. Corporate Office : Aakash Tower, 8, Pusa Road, New Delhi-110005. Phone : 011-47623456 Solutions of Assignment (Set-1) (Level!) System of Particles and Rotational Motion 67 51. We know that when an object of finite size is thrown with some intial velocity at an angle with the horizontal, it follows a parabolic path. The centre of mass of such an object also follows the parabolic path, even if the object were to disintegrate in mid-air. For example, when a fire cracker projected from O explodes in mid-air, the fragments fly off in different directions, describing their own parabolic paths. However, the centre of mass of the cracker would stil continue to move along the same parabolic path as shown in the figure given below. y Parabolic path of the proje Explosion Path of the CM of fragments. Fig. The centre of mass (CM) of the fragments of the projectile continues along the same parabolic path which it would have followed if there were no explosion 52. Angular displacement (8) is the angle described bythe posion vector 7 about the axis of cation. ae a Angular acceleration (ct) of a rigid body is the rate of change of angular velocity of the body about the given ao at Relation between angular momentum and torque Angular velocity (a) is the rate at which angular displacement changes with time i.e. axis of rotation i.e. dj 5) _ + dP 1B) x at Veyinsee ‘Therefore solid sphere reaches the bottom first. Corporate Office : Aakash Tower, 8, Pusa Road, New Delhi-110005. Phone : 011-47623456 Solutions of Assignment (Set-1) (Level!) 56. System of Particles and Rotational Motion 69) (i) Theorems of perpendicular axis for moment of inertia of a planar body states that Itty where fly and J, ate the moment of inertia ofthe body about three mutually perpendicular axis — x-axis, yraxis and z-axis respectively. Here x and y-axis are inthe plane of body and z-axis is perpendicular to the plane of body. The three axis intersect ata point. This theorems applicable for planar bodies only (ii) In a perfectly rigid body there is no internal motion therefore, the work done by the external torques is not dissipated and goes on to increase the kinetic energy of the body. The rate of increase of KE. is given d “ike by G (Ke) Nig? KE, of rotation motion KE, = 10 A(la") _ 2a do ala) =! ya Hero, we assume that moment of inertia does not change with time. This means tho mass of the body does not change. The body remains rigid and also the axis does not change fe., is postion wth respect to the body remains the same. de dt (10°) say dt a Change in kinetic energy is due to work done, and the rate of change of kinetic eneray is equal to the rate at which the work is done, by the extemal torque which given by tw. So on equality the rate of work done and rate of increase of KE, we get ww = foo. SECTION - B Answer (3) ‘Angle between 7 and & is 90° Hence v= ro @= = 8 rad/s Answer (3) 1 civen IAxBl- AB v3 ABsind = ABcoso V3 Corporate Office : Aakash Tower, 8, Pusa Road, New Delhi-110005. Phone : 011-47623456 70 System of Particles and Rotational Motion Solutions of Assignment (Set-1) (Level) ” 2, 8. 14, 45. tano= 4 V3 0-30° Now, [A -Bl= VA? B® 2ABcos0 =(0? +8 ABN)F Answer (2) 1 lag + me here d= R mR? + mR? 2mR? Answer (2) Pye = 11.0088 = 10x30xcos0" = 300 W Answer (1) Aga? - 2x tmv? 2 z ve 2 so, body is solid sphere, axb+axé Couple Large ‘The dimensions of angular momentum is [M'L2T-"]. Its unit is kg m2s~', It is a vector quantity, at Torque (*)= $F The centre of mass will be closer to the heavier particle. ‘The centre of mass of a uniform circular ring lies at the centre of the ring. Moment of inertia of a body depends on the (Axis of rotation. (i) Mass of the body. Angular velocity (@) decreases because angular momentum is conserved L = fi, On stretching hands | increases hence, w decreases. Because the entire mass of the ring is at its periphery i. at the maximum distance from the centre where as the mass of the disc is distributed from the centre to the rim, Corporate Office : Aakash Tower, 8, Pusa Road, New Delhi-110005. Phone : 011-47623456 Solutions of Assignment (Set-1) (Level-) System of Particles and Rotational Motion 711 16. L= Ja = constant is decreased, so w increases KE, = Lie? rotation = as w increases, K.E. increases, hence the kinetic energy of the rotation of the system will increase. 47. According to the theorem of perpendicular axes, the moment of inertia of a planar body (lamina) about an axis perpendicular to its plane is equal to the sum of its moments of inertia about two perpendicular axes concurrent with perpendicular axis and lying in the plane of the body. 18. =i(-3+3)-j(2-1)+k(-6+3) =0/ -j-3k 19. Hence, the angular momentum of the patticle remains the same. 2nx100_ 20 6 6 20. w= 0, @, = 100 rp.m. a=2rads%1= 7,8 Using w= @, + at 20, Or 2 6 a =2 Using, a? - of = 200 (222) -2x20 400? 100 6 326x436 In one rotation, the angular displacement is 2n- 100n* 1 _ 50 Number of rotations = “7 x > = Fe Corporate Office : Aakash Tower, 8, Pusa Road, New Delhi-110005. Phone : 011-47623456 72. System of Particles and Rotational Motion Solutions of Assignment (Set-1) (Level) 21 A 8. ze saa Dae a ABC is an equilateral triangle, axis of rotation is AD ‘m,lies on the axis of rotation ie. AD. r= 0} |= 2 (m, +m) 22. The angular displacement of the ring in 1 revolution = 27. So, the angular displacement of the ring in 20 revolutions is 20 x 2x = 40x Using, @ 03 = 208 . = 2200 40 gos 8 = 40x, o, = 200 pm = 2=E00 - Barraas o=0 a=? Nogatvo sign show thatthe ring is slowing down «= Moment of neta of he ing about an ais passing Hough its daetris given by = MA = 94g, R= 20 em 30, 0 8, 700 “100 100 = 0.1047 Nm 23, Corporate Office : Aakash Tower, 8, Pusa Road, New Delhi-110005. Phone : 011-47623456 Solutions of Assignment (Set-t) (Levelt) System of Particles and Rotational Motion 73, 24, 28. 26. (-9+9)7-j(6-6)+k (18-18) 0 Since the cross product of A and B is zero, Therefore, these are parallel vectors 4 Rotational energy KE. = 5/4 4 nyt ranslational K.E = mv’ Transtational KE = > Inv? + Ne? Total energy = 57v* + 5a" for solid cylinder 1 SoKE,, So the total KE (The ratio of rotational energy to total energ (i) The ratio of translation energy to rotational energy = 2 4 While falling a cat stretches its body along with the tall so that its moment of inertia (/) increases. As no external torque is acting therefore, its angular momentum is conserved i.e. L = luv = constant, as / increases, @ decreases and this helps it to land gently on its feet, (Ifthe external forces acting on the system of particles is zero => ta1=0, hence di ° at or [= constant Thus, if the total external torque on a system of particles is zero, then the total angular momentum of the system is conserved, i) Corporate Office : Aakash Tower, 8, Pusa Road, New Delhi-110005. Phone : 011-47623456 74 System of Particles and Rotational Motion Solutions of Assignment (Set-1) (Level) - dB. So, 7x2 = 0, Fx = TXE Hence, &(FxP)=i di_- and or OR () Torque (2): A pair of equal and opposite forces with diferent ines of action is known as a couple or torque. A couple produces rotation without translation. It can also be defined asthe rate of change of a at angular momentum of a rigid body i.e. (il) In a perfectly rigid body there is no internal motion therefore, the work done by the external torques is not dissipated and goes on to increase the kinetic eneray of the body. The rate of increase of KE. is d given by (KE) Vie? K.E. of rotational motion K.E. = 5/0 d (lw) 20de ata) =! Fae Here, we assume that moment of inertia does not change with time. This means the mass of the body does not change. The body remains rigid and also the axis does not change i.e., its position with respect to the body remains the same. oe a (ta), ala )o Change in kinetic energy is due to work done, and the rate of change of kinetic energy is equal to the rate at which the work is done, by the extemal torque which is given by to. So on equality, the rate of work done and rate of increase of K.E., we get a= loo, [e= Tal 27. According to the theorem of parallel axis. The moment of inertia of a body about any axis is equal to the sum of the moment of inertia of the body about a parallel axis passing through its centre of mass and the product of its mass and the square of the distance between the two parallel axis, <—_12—__ > Corporate Office : Aakash Tower, 8, Pusa Road, New Delhi-110005. Phone : 011-47623456 Solutions of Assignment (Set-1) (Level!) System of Particles and Rotational Motion 75) ‘The moment of inertia (/) about an axis passing through O. tay = EY) 2 ‘The moment of inertia (I) about an axis passing through O. oR Moment of Inertia : The moment of inertia of a rigid body about a given axis of rotation is the sum of the sular distances from the axis of products of masses of the various particles and squares of their perpen« rotation t= me moment of inertia of a rod about an axis passing through its centre of mass is Now, rod is bent to form a hexagon of side a. A hexagon is formed by equilateral triangles. v3 So, consider ABC altitude, 8D 2 which is the distance of the centre of mass of hexagon from the GM. of tho rod AC. The mass of each sid of Hexagon Is 1 kg an moment oferta is 2, moment of inertia of one side about the centre of mass of the hexagon is given by using the parallel axis theorem ma’ (J3_\’_ ma? _ 3ma* ta ema) SE tee _ ma? +9ma?_10 2 2 m= 1 kg, 1 0 leg = ae $0 hy = 45 ‘i i8 the moment of inertia of one side about an axis passing C.M. of hexagon, So total moment of inertia of hexagon about this axis is given by 10, 12 So the moment of inertia of cylinder about an axis passing through the centre of mass Is 5a. 6x a? = Set aaa Corporate Office : Aakash Tower, 8, Pusa Road, New Delhi-110005. Phone : 011-47623456 76 System of Particles and Rotational Motion Solutions of Assignment (Set-2) (Level) Solutions (Set-2) SECTION -A 1. Answer (3) yk + Hpk + Mg ky 7, +m, + (19)(0}+(24)(0}+2(12) 94, 1642492 _(1.6}(0] +(2.4)1+2(0) ™ 1.642442 $0, (Yq) = (014, 014) m 2. Answer (3) Centre of mass of the uniform rod will ie at its centre MX m+ My (L) oni) x ha) tame) yg nl) m0) em 3m om om cents yg 2h en = B+ Ven = 3. Answer (2) = X,, is inside the square plate Corporate Office : Aakash Tower, 8, Pusa Road, New Delhi-110005. Phone : 011-47623456 Solutions of Assignment (Set-2) (Level) System of Particles and Rotational Motion 77 4, Answer (2) Centre of mass wil lie on the line of symmetry A OA is the line of symmetry of the remaining part 5. Answer (2) Net external force is zero, and centre of mass of the system is intially at rest. So position of centre of mass will not change. So to have x,,, = constant the balloon will move downwards. 6. Answer (4) 7. Answer (4) IF. =0 V,, iS at rest initially so v,, = 0 asF,=0 = a,,=0 8. Answer (4) The path of centre of mass will not change due to internal forces 9. Answer (1) m(-9}+m(-9) _m{cg)+mi-9) | Bem = em mem 10. Answer (3) (4) (20 20-2(2)(2\(22}n [ler impulse L2})G00: ‘Change in angular momentum = 500 - 250 rads 2 11. Answer (2) Use «= fa 10. 2,y( 10)? 2 (29) eee 450 rae? 20(Goa)~3Gop) & > a= 480 12. Answer (3) Corporate Office : Aakash Tower, 8, Pusa Road, New Delhi-110005. Phone : 011-47623456 78 System of Particles and Rotational Motion Solutions of Assignment (Set-2) (Level) 13. 14, 15. 16. 17. 18, Answer (2) ww, at Wy _ 2n(240) t 60x60 a= adie? 60 a= = Iu 8n 60 ‘Answer (4) 7 Ff = (21+4]+7k) [i +2)+3k) is2j+ak F=2i+3]-5k tah xF yi « 2 4] =-2oi+19)-% bk a-s Answer (1) ‘Answer (2) ot L=me of =(6}(32)(2s3) = 60 units Answer (3) L= mv, 1, fs constant 80 L = constant Answer (2) Net toraue = 0 for equilibrium raen( (3 mi 121-45) = 1-451-4 75-0 2 m= 15 kg Corporate Office : Aakash Tower, 8, Pusa Road, New Delhi-110005. Phone : 011-47623456 Solutions of Assignment (Set-2) (Level-) ‘System of Particles and Rotational Motion 79) 19. Answer (4) me 20. Answer (4) Now perpendicular axis theorem rerer 2 22. Answer (1) Iron is much denser than Aluminium. To have the maximum moment of inertia, material having higher density should be placed farther from the rotational axis. 23. Answer (3) 24, Answer (4) (2 ape?) + 2mb? a( Zima?) +200 ~ Sma? + 2Mb2 5 Corporate Office : Aakash Tower, 8, Pusa Road, New Delhi-110005. Phone : 011-47623456 80 System of Particles and Rotational Motion Solutions of Assignment (Set-2) (Level) 25. Answer (2) Using parallel axis theorem for one rod mf (0 52 ™zp) For all three rods mit = 3) =3mi? am rear > a lat 26. Answer (4) Using perpendicular axis theorem, /# 1, +f, +1,+ |, 27. Answer (4) 28. 29. Answer (3) 1 = 0, + at Now, 0= at +5 at 1 20 = 0+ a(10) = 5 (2) (100) a= 2 radis? = 100 radian Corporate Office : Aakash Tower, 8, Pusa Road, New Delhi-110005. Phone : 011-47623456 Solutions of Assignment (Set-2) (Level-) 30. Answer (2) 31 32, Answer (3) Sms me Lele 20 ! 2 xsxt=0020 3 w= 2: 7500 8% p= 00 = 25 radis 33. Answer (2) k=dinv? + 2 h0? 2 a Amr v2 =f tt Baya? Seay FV) <6 34, Answer (2) vy, Vast = Wem * Vim Vem V2 Corporate Office : Aakash Tower, 8, Pusa Road, New Del System of Particles and Rotational Motion 4110005. Phone : 011-47623456 81 82 System of Particles and Rotational Motion 35. Answer (3) 10,= 10, Mee = (M + 2m) at or Me Mom 36. Answer (3) 0, = 220 _ 35 ips m= = Ses 2n60 ae = 2nps Using angular momentum conservation 11.5) = (I+ me) (1) Ly? fe mr’ 2 1 2mm? 37. Answer (2) Using angular momentum conservation, I, + La, =(I, * Io Loy hey hth 38. Answer (4) Let Solutions of Assignment (Set-2) (Level) Corporate Office : Aakash Tower, 8, Pusa Road, New Delhi-110005. Phone : 011-47623456 Solutions of Assignment (Set-2) (Level-) System of Particles and Rotational Motion 39. Answer (3) 40. Answer (1) rodt At thnk 3h 7a 41, Answer (3) 42, Answer (1) <0.75 m—> 0.75 m—> A B > 20N 30N Net torque should be same for the new point 20(0.75) + 30(0.75) = 50(x) Solve for x 43, Answer (1) Ke Body of smaller will take loss time so solid sphere will reach the ground frst. 44, Answer (3) Using mechanical energy conservation 1) ev (Qua tiyeat gh = Amv? +4 m3 2 Corporate Office : Aakash Tower, 8, Pusa Road, New Delhi-110005. Phone : 011-47623456 83 84 System of Particles and Rotational Motion Solutions of Assignment (Set-2) (Level) 45. Answer (1) 46. Answer (2) 47. Answer (2) 1 12 Mgh = *mv? 44 igh=> zl 4 pyy2 4 Lmi® v2 48, Answer (4) ¢__mgrsin OCT Zine? met a ar - mar?sine Zine mr Corporate Office : Aakash Tower, 8, Pusa Road, New Delhi-110005. Phone : 011-47623456 Solutions of Assignment (Set-2) (Level-) System of Particles and Rotational Motion 49, 50. 10. " Answer (2) a Answer (1) Using angular momentum conservation, by pulling his arms and legs in, moment of inertia will decrease hence « will increase SECTION -B Answer (3) ‘Answer (1) Answer (3) = Mi+j)+2(2i+5])+4(2—j) am _ is jeai+toj+ei—aj 131477 7 Answer (4) ta= 12105 25008 t Answer (2) If a particle moves with constant velocity then its angular momentum remain constant. Answer (3) Sinyt Ato? Lim? etn 2met a? = bmw? + Lia? = dmv? + mnie’ t= fon? hat = hmv + Sick = Nant Nyy? = SE 2 3 “6 Answer (3) Answer (3) ‘Answer (3) 0%2+0x242N8 _ 2/31 Yon 6 6 We Answer (2) a= 8 t = 20 AR raais? Ba 8 ‘Answer (4) Corporate Office : Aakash Tower, 8, Pusa Road, New Del 4110005. Phone : 011-47623456 85 86 System of Particles and Rotational Motion 2, 13. 14. 15. 16. 7 18, Solutions of Assignment (Set-2) (Level) Pow = 100x108 = 1x250 = 1400 Nem Answer (3) ‘Answer (1) Answer (4) Answer (1) me 12 me mK? = MA 12 L eke Answer (3) By conservation of angular momentum mv = lo Bm+M)Co_ 3 o- 3m L(3m+M) Answer (1) @=3-5t mv. Answer (2) 2MR? _2MR? 5 on 2 \ 2 aT By conservation of angular momentum. Corporate Office : Aakash Tower, 8, Pusa Road, New Delhi-110005. Phone : 011-47623456 Solutions of Assignment (Set-2) (Level-) System of Particles and Rotational Motion 2MR? 2m _2 Bo “7, =T 19. Answer (3) AxB = ABsinoi =[AxBlA ._ AxB fiz lax) 20. Answer (2) The velocity of centre of mass will be zero, 4% (10-v)=40 xv = 40-4v=40v = 4av=40 10 > v= yyms 21. Answer (4) 2. 23, (OA = Scos45° 39 Lem =1x3x 3-2 kgm 2 v2 24, Answer (2) By conservation of angular momentum. Jo =2le, Corporate Office : Aakash Tower, 8, Pusa Road, New Del 4110005. Phone : 011-47623456 87 88 System of Particles and Rotational Motion 1 Ka pal 25, Answer (1) 2, ME _ Me MOE 40 Lit Teo iS 26. Answer (4) 27. Answer (1) 2u? sindcos6 2g c=mgx = r=mu*sinecose Solutions of Assignment (Set-2) (Level) Corporate Office : Aakash Tower, 8, Pusa Road, New Delhi-110005. Phone : 011-47623456

You might also like